Peace Yow Flashcards
1. A 53-year-old woman is found, by arteriography, to have 50% narrowing of her left renal artery. What is the expected change in blood flow through the stenotic artery? (A) Decrease to ½ (B) Decrease to ¼ (C) Decrease to ⅛ (D) Decrease to 1⁄16 (E) No change
- When a person moves from a supine position to a standing position, which
of the following compensatory changes occurs?
(A) Decreased heart rate
(B) Increased contractility
(C) Decreased total peripheral resistance (TPR)
(D) Decreased cardiac output
(E) Increased PR intervals
- When a person moves from a supine position to a standing position, which
of the following compensatory changes occurs?
(A) Decreased heart rate
(B) Increased contractility
(C) Decreased total peripheral resistance (TPR)
(D) Decreased cardiac output
(E) Increased PR intervals
3. At which site is systolic blood pressure the highest? (A) Aorta (B) Central vein (C) Pulmonary artery (D) Right atrium (E) Renal artery (F) Renal vein
- A person’s electrocardiogram (ECG) has no P wave but has a normal QRS
complex and a normal T wave. Therefore, his pacemaker is located in the
(A) sinoatrial (SA) node
(B) atrioventricular (AV) node
(C) bundle of His
(D) Purkinje system
(E) ventricular muscle
5. If the ejection fraction increases, there will be a decrease in (A) cardiac output (B) end-systolic volume (C) heart rate (D) pulse pressure (E) stroke volume (F) systolic pressure
An electrocardiogram (ECG) on a person shows ventricular extrasystoles.
6. The extrasystolic beat would produce
(A) increased pulse pressure because contractility is increased
(B) increased pulse pressure because heart rate is increased
(C) decreased pulse pressure because ventricular filling time is increased
(D) decreased pulse pressure because stroke volume is decreased
(E) decreased pulse pressure because the PR interval is increased
- After an extrasystole, the next “normal” ventricular contraction produces
(A) increased pulse pressure because the contractility of the ventricle is
increased
(B) increased pulse pressure because total peripheral resistance (TPR) is
decreased
(C) increased pulse pressure because compliance of the veins is decreased
(D) decreased pulse pressure because the contractility of the ventricle is
increased
(E) decreased pulse pressure because TPR is decreased
- An increase in contractility is demonstrated on a Frank-Starling diagram by
(A) increased cardiac output for a given end-diastolic volume
(B) increased cardiac output for a given end-systolic volume
(C) decreased cardiac output for a given end-diastolic volume
(D) decreased cardiac output for a given end-systolic volume
12. If the heart rate is 70 beats/min, then the cardiac output of this ventricle is closest to (A) 3.45 L/min (B) 4.55 L/min (C) 5.25 L/min (D) 8.00 L/min (E) 9.85 L/min
In a capillary, Pc is 30 mm Hg, Pi is −2 mm Hg, πc is 25 mm Hg, and πi is 2 mm Hg. 13. What is the direction of fluid movement and the net driving force? (A) Absorption; 6 mm Hg (B) Absorption; 9 mm Hg (C) Filtration; 6 mm Hg (D) Filtration; 9 mm Hg (E) There is no net fluid movement
14. If Kf is 0.5 mL/min/mm Hg, what is the rate of water flow across the capillary wall? (A) 0.06 mL/min (B) 0.45 mL/min (C) 4.50 mL/min (D) 9.00 mL/min (E) 18.00 mL/min 15. The tendency for blood flow to be turbulent is increased by (A) increased viscosity (B) increased hematocrit (C) partial occlusion of a blood vessel (D) decreased velocity of blood flow
- A 66-year-old man, who has had a sympathectomy, experiences a greater-
than-normal fall in arterial pressure upon standing up. The explanation for
this occurrence is
(A) an exaggerated response of the renin–angiotensin–aldosterone system
(B) a suppressed response of the renin–angiotensin–aldosterone system
(C) an exaggerated response of the baroreceptor mechanism
(D) a suppressed response of the baroreceptor mechanism
17. The ventricles are completely depolarized during which isoelectric portion of the electrocardiogram (ECG)? (A) PR interval (B) QRS complex (C) QT interval (D) ST segment (E) T wave 18. In which of the following situations is pulmonary blood flow greater than aortic blood flow? (A) Normal adult (B) Fetus (C) Left-to-right ventricular shunt (D) Right-to-left ventricular shunt (E) Right ventricular failure (F) Administration of a positive inotropic agent
- A 30-year-old female patient’s electrocardiogram (ECG) shows two P
waves preceding each QRS complex. The interpretation of this pattern is
(A) decreased firing rate of the pacemaker in the sinoatrial (SA) node
(B) decreased firing rate of the pacemaker in the atrioventricular (AV) node
(C) increased firing rate of the pacemaker in the SA node
(D) decreased conduction through the AV node
(E) increased conduction through the His–Purkinje system
- An acute decrease in arterial blood pressure elicits which of the following
compensatory changes?
(A) Decreased firing rate of the carotid sinus nerve
(B) Increased parasympathetic outflow to the heart
(C) Decreased heart rate
(D) Decreased contractility
(E) Decreased mean systemic filling pressure - The tendency for edema to occur will be increased by
(A) arteriolar constriction
(B) increased venous pressure
(C) increased plasma protein concentration
(D) muscular activity
- Inspiration “splits” the second heart sound because
(A) the aortic valve closes before the pulmonic valve
(B) the pulmonic valve closes before the aortic valve
(C) the mitral valve closes before the tricuspid valve
(D) the tricuspid valve closes before the mitral valve
(E) filling of the ventricles has fast and slow components - During exercise, total peripheral resistance (TPR) decreases because of the
effect of
(A) the sympathetic nervous system on splanchnic arterioles
(B) the parasympathetic nervous system on skeletal muscle arterioles
(C) local metabolites on skeletal muscle arterioles
(D) local metabolites on cerebral arterioles
(E) histamine on skeletal muscle arterioles
- An increase in arteriolar resistance, without a change in any other
component of the cardiovascular system, will produce
(A) a decrease in total peripheral resistance (TPR)
(B) an increase in capillary filtration
(C) an increase in arterial pressure
D) a decrease in afterload
28.The following measurements were obtained in a male patient:
Central venous pressure: 10 mm Hg
Heart rate: 70 beats/min
Systemic arterial [O2] = 0.24 mL O2/mL
Mixed venous [O2] = 0.16 mL O2/mL
Whole body O2 consumption: 500 mL/min
What is this patient’s cardiac output?
(A) 1.65 L/min
(B) 4.55 L/min
(C) 5.00 L/min
(D) 6.25 L/min
(E) 8.00 L/min
29. Which of the following is the result of an inward Na+ current?
(A) Upstroke of the action potential in the sinoatrial (SA) node
(B) Upstroke of the action potential in Purkinje fibers
(C) Plateau of the action potential in ventricular muscle
(D) Repolarization of the action potential in ventricular muscle
(E) Repolarization of the action potential in the SA node
- The greatest pressure decrease in the circulation occurs across the
arterioles because
A) they have the greatest surface area
(B) they have the greatest cross-sectional area
(C) the velocity of blood flow through them is the highest
(D) the velocity of blood flow through them is the lowest
(E) they have the greatest resistance - Pulse pressure is
(A) the highest pressure measured in the arteries
(B) the lowest pressure measured in the arteries
(C) measured only during diastole
(D) determined by stroke volume
(E) decreased when the capacitance of the arteries decreases
(F) the difference between mean arterial pressure and central venous pressure
- In the sinoatrial (SA) node, phase 4 depolarization (pacemaker potential) is
attributable to
(A) an increase in K+ conductance
(B) an increase in Na+ conductance
(C) a decrease in Cl− conductance
(D) a decrease in Ca2+ conductance
(E) simultaneous increases in K+ and Cl− conductances - A healthy 35-year-old man is running a marathon. During the run, there is
an increase in his splanchnic vascular resistance. Which receptor is
responsible for the increased resistance?
(A) α1 Receptors
(B) β1 Receptors
(C) β2 Receptors
(D) Muscarinic receptors
- During which phase of the cardiac cycle is aortic pressure highest?
(A) Atrial systole
(B) Isovolumetric ventricular contraction
(C) Rapid ventricular ejection
(D) Reduced ventricular ejection
(E) Isovolumetric ventricular relaxation
(F) Rapid ventricular filling
(G) Reduced ventricular filling (diastasis)
37. Myocardial contractility is best correlated with the intracellular concentration of (A) Na+ (B) K+ (C) Ca2+ (D) Cl− (E) Mg2+ 38. Which of the following is an effect of histamine? (A) Decreased capillary filtration (B) Vasodilation of the arterioles (C) Vasodilation of the veins (D) Decreased Pc (E) Interaction with the muscarinic receptors on the blood vessels
39. Carbon dioxide (CO2) regulates blood flow to which one of the following organs? (A) Heart (B) Skin (C) Brain (D) Skeletal muscle at rest (E) Skeletal muscle during exercise 40. Cardiac output of the right side of the heart is what percentage of the cardiac output of the left side of the heart? (A) 25% (B) 50% (C) 75% (D) 100% (E) 125%
- The physiologic function of the relatively slow conduction through the
atrioventricular (AV) node is to allow sufficient time for
(A) runoff of blood from the aorta to the arteries
(B) venous return to the atria
(C) filling of the ventricles
(D) contraction of the ventricles
(E) repolarization of the ventricles
- The physiologic function of the relatively slow conduction through the
atrioventricular (AV) node is to allow sufficient time for
(A) runoff of blood from the aorta to the arteries
(B) venous return to the atria
(C) filling of the ventricles
(D) contraction of the ventricles
(E) repolarization of the ventricles
42. Blood flow to which organ is controlled primarily by the sympathetic nervous system rather than by local metabolites? (A) Skin (B) Heart (C) Brain (D) Skeletal muscle during exercise 43. Which of the following parameters is decreased during moderate exercise? (A) Arteriovenous O2 difference (B) Heart rate (C) Cardiac output (D) Pulse pressure (E) Total peripheral resistance (TPR)
- A 72-year-old woman, who is being treated with propranolol, finds that she
cannot maintain her previous exercise routine. Her physician explains that
the drug has reduced her cardiac output. Blockade of which receptor is
responsible for the decrease in cardiac output?
(A) α1 Receptors
(B) β1 Receptors
(C) β2 Receptors
(D) Muscarinic receptors
(E) Nicotinic receptors - During which phase of the cardiac cycle is ventricular volume lowest?
(A) Atrial systole
(B) Isovolumetric ventricular contraction
(C) Rapid ventricular ejection
(D) Reduced ventricular ejection
(E) Isovolumetric ventricular relaxation
(F) Rapid ventricular filling
(G) Reduced ventricular filling (diastasis) - Which of the following changes will cause an increase in myocardial O2
consumption?
(A) Decreased aortic pressure
(B) Decreased heart rate
(C) Decreased contractility
(D) Increased size of the heart
(E) Increased influx of Na+ during the upstroke of the action potential
- Which of the following substances crosses capillary walls primarily
through water-filled clefts between the endothelial cells?
(A) O2
(B) CO2
(C) CO
(D) Glucose - A 24-year-old woman presents to the emergency department with severe
diarrhea. When she is supine (lying down), her blood pressure is 90/60 mm
Hg (decreased) and her heart rate is 100 beats/min (increased). When she is
moved to a standing position, her heart rate further increases to 120
beats/min. Which of the following accounts for the further increase in heart
rate upon standing?
(A) Decreased total peripheral resistance
(B) Increased venoconstriction
(C) Increased contractility
(D) Increased afterload
(E) Decreased venous return
- A 60-year-old businessman is evaluated by his physician, who determines
that his blood pressure is significantly elevated at 185/130 mm Hg.
Laboratory tests reveal an increase in plasma renin activity, plasma
aldosterone level, and left renal vein renin level. His right renal vein renin
level is decreased. What is the most likely cause of the patient’s
hypertension?
(A) Aldosterone-secreting tumor
(B) Adrenal adenoma secreting aldosterone and cortisol
(C) Pheochromocytoma
(D) Left renal artery stenosis
(E) Right renal artery stenosis
- Propranolol has which of the following effects?
A) Decreases heart rate
- During which phase of the cardiac cycle does the mitral valve open?
(A) Atrial systole
(B) Isovolumetric ventricular contraction
(C) Rapid ventricular ejection
(D) Reduced ventricular ejection
(E) Isovolumetric ventricular relaxation
(F) Rapid ventricular filling
(G) Reduced ventricular filling (diastasis)
(E) Decreased atrial natriuretic peptide levels, decreased Na+ reabsorption
- A hospitalized patient has an ejection fraction of 0.4, a heart rate of 95
beats/min, and a cardiac output of 3.5 L/min. What is the patient’s end-
diastolic volume?
D) 92 mL
First, calculate stroke volume from the cardiac
output and heart rate: Cardiac output = stroke volume × heart rate; thus, stroke
volume = cardiac output/heart rate = 3500 mL/95 beats/min = 36.8 mL. Then,
calculate end-diastolic volume from stroke volume and ejection fraction:
Ejection fraction = stroke volume/end-diastolic volume; thus, end-diastolic
volume = stroke volume/ejection fraction = 36.8 mL/0.4 = 92 mL.
- A 38-year-old woman has a bout of “intestinal flu,” with vomiting and
diarrhea for several days. Although she is feeling better, when she stands up
quickly, she feels faint and light-headed. Which of the following explains why she is light-headed?
(A) Decreased blood volume, decreased preload, decreased cardiac output
The woman has significant loss of extracellular
fluid (and blood) volume due to vomiting and diarrhea. Decreased blood volume leads to decreased venous return, decreased preload, and decreased cardiac output by the Frank-Starling mechanism; the decrease in cardiac output causes decreased arterial pressure (Pa) and decreased cerebral blood flow, which is responsible for the feeling of light-headedness. The decrease in Pa will activate
both the baroreceptor mechanism and the renin–angiotensin II–aldosterone system, but the results of turning on these mechanisms (increased sympathetic
output, increased total peripheral resistance, increased heart rate, and compensatory increase in Pa toward normal) are secondary to the decrease in Pa;they are not causes of the light-headedness. Likewise, decreased atrial natriuretic peptide levels and decreased Na+ reabsorption can occur secondary to the
decrease in Pa but are not causes of the light-headedness.
- Which agent is released or secreted after a hemorrhage and causes an
increase in renal Na+ reabsorption?
A) Aldosterone
Angiotensin I and aldosterone are
increased in response to a decrease in renal perfusion pressure. Angiotensinogen is the precursor for angiotensin I. Antidiuretic hormone (ADH) is released when atrial receptors detect a decrease in blood volume. Of these, only aldosterone increases Na+ reabsorption. Atrial natriuretic peptide is released in response to an increase in atrial pressure, and an increase in its secretion would not be anticipated after blood loss.
- The low-resistance pathways between myocardial cells that allow for the
spread of action potentials are the
A) gap junctions
The gap junctions occur at the intercalated disks
between cells and are low-resistance sites of current spread.
- Which receptor mediates slowing of the heart?
(D) Muscarinic receptors
Acetylcholine (ACh) causes slowing
of the heart via muscarinic receptors in the sinoatrial (SA) node.
- Which of the following agents or changes has a negative inotropic effect on
the heart?
(A) Increased heart rate
A negative inotropic effect is one that decreases
myocardial contractility. Contractility is the ability to develop tension at a fixed
muscle length. Factors that decrease contractility are those that decrease the
intracellular [Ca2+]. Increasing heart rate increases intracellular [Ca2+] because
more Ca2+ ions enter the cell during the plateau of each action potential.Sympathetic stimulation and norepinephrine increase intracellular [Ca2+] by increasing entry during the plateau and increasing the storage of Ca2+ by the sarcoplasmic reticulum (SR) [for later release]. Cardiac glycosides increase intracellular [Ca2+] by inhibiting the Na+–K+ pump, thereby inhibiting Na+–Ca2+
exchange (a mechanism that pumps Ca2+ out of the cell). Acetylcholine (ACh)has a negative inotropic effect on the atria.